Wight

New Member
ارسال ها
1,188
لایک ها
888
امتیاز
0
#1
این تاپیک برای حل مسائل مربوط به دنباله ها و مسائل مربوط به آن ایجاد شده.

لطفا همکاری های لازم را ! با ما بفرمایید!!!
 
آخرین ویرایش توسط مدیر

amin

New Member
ارسال ها
312
لایک ها
515
امتیاز
0
#2
پاسخ : دنباله

خوب این سوال کلا غلطه چون:

پس جواب بین (1,2) هست که غلطه.
 

Wight

New Member
ارسال ها
1,188
لایک ها
888
امتیاز
0
#3
پاسخ : دنباله

خوب این سوال کلا غلطه چون:

پس جواب بین (1,2) هست که غلطه.
بله به این جواب رسیدم !!!! ولی کلا اگه جواب داشت و مثلا جوابش 40 بود!! راه حل چی میشه!
 

Al!R3ZA

Well-Known Member
ارسال ها
1,903
لایک ها
3,166
امتیاز
113
#4
پاسخ : دنباله

بله به این جواب رسیدم !!!! ولی کلا اگه جواب داشت و مثلا جوابش 40 بود!! راه حل چی میشه!
اگر فرض کنیم سوال ، جوابی مثل m داشته باشه که توش صدق کنه ،
دنبالتون رو تجزیه میکنید ، میشه :

حالا واضحه که m عدد طبیعیه ، از اون طرف n هم طبیعیه ( چون دنباله است ) پس اون دوتا پرانتز هم طبیعی ان ( اگه n یک نباشه که نیست ! ) پس اون سه ، مقسوم علیه های m هستن.
m رو تجزیه میکنید ، با یکم فکر کردن ، میتونید اون اعداد تجزیه شده رو به صورت اون سه ضرب سمت چپ بنویسید.
همین ! :4::p
(دقت کنید اون دوتا پرانتز رو تجزیه نکردم چون راحت تر میشه دوتا مضرب رو پیدا کرد که اختلافشون 2 که همون اختلاف اون دو تا پرانتزه رو پیدا کرد ! )
 

amin

New Member
ارسال ها
312
لایک ها
515
امتیاز
0
#5
پاسخ : دنباله

راه اقا Al!R3ZA عالیه
منم یه نظری دارم البته درست نیست و تقریبی میشه حساب کرد اونم با ماشین حساب
با توجه به

می توان دریافت که مقدار اعداد با شتاب زیادی داره افزایش پیدا می کنه پس
تاثیر زیادی نخواهد داشت
پس

فقط قسمت اعشار رو حذف می کنیم و قسمت صحیح رو به علاوه ی 1 می کنیم


 

Al!R3ZA

Well-Known Member
ارسال ها
1,903
لایک ها
3,166
امتیاز
113
#6
پاسخ : دنباله

راه اقا Al!R3ZA عالیه
منم یه نظری دارم البته درست نیست و تقریبی میشه حساب کرد اونم با ماشین حساب
با توجه به

می توان دریافت که مقدار اعداد با شتاب زیادی داره افزایش پیدا می کنه پس
تاثیر زیادی نخواهد داشت
پس

فقط قسمت اعشار رو حذف می کنیم و قسمت صحیح رو به علاوه ی 1 می کنیم


اینم راه جالبی بود. حد خوندی؟
البته فک نکنم سر جلسات امتحان ( امتحانایی مثل کنکور و شبیهش ) اجازه بدن ماشین حساب ببرین :160:
 

P3YM4N

New Member
ارسال ها
70
لایک ها
63
امتیاز
0
#7
پاسخ : دنباله

اگر فرض کنیم سوال ، جوابی مثل m داشته باشه که توش صدق کنه ،
دنبالتون رو تجزیه میکنید ، میشه :

حالا واضحه که m عدد طبیعیه ، از اون طرف n هم طبیعیه ( چون دنباله است ) پس اون دوتا پرانتز هم طبیعی ان ( اگه n یک نباشه که نیست ! ) پس اون سه ، مقسوم علیه های m هستن.
m رو تجزیه میکنید ، با یکم فکر کردن ، میتونید اون اعداد تجزیه شده رو به صورت اون سه ضرب سمت چپ بنویسید.
همین ! :4::p
(دقت کنید اون دوتا پرانتز رو تجزیه نکردم چون راحت تر میشه دوتا مضرب رو پیدا کرد که اختلافشون 2 که همون اختلاف اون دو تا پرانتزه رو پیدا کرد ! )
سلام
من نتوستم برای این معادله جواب طبیعی پیدا کنم:181:
درست متوجه نمیشم
مگه نباید n عدد طبیعی باشه؟
میشخ بیشتر بتوضیحید
ممنون;)
 

Wight

New Member
ارسال ها
1,188
لایک ها
888
امتیاز
0
#8
پاسخ : دنباله

سلام
من نتوستم برای این معادله جواب طبیعی پیدا کنم:181:
درست متوجه نمیشم
مگه نباید n عدد طبیعی باشه؟
میشخ بیشتر بتوضیحید
ممنون;)
بله این سوال اشتباه تایپ شده بود! رفتم پرسیدم گفتند n^2 - n که جواب می شه 9!
 

P3YM4N

New Member
ارسال ها
70
لایک ها
63
امتیاز
0
#9
پاسخ : دنباله

بله این سوال اشتباه تایپ شده بود! رفتم پرسیدم گفتند n^2 - n که جواب می شه 9!
آخیش مطمن شدم
داشتم به دانسته های خودم شک میکردم:d
 

amin

New Member
ارسال ها
312
لایک ها
515
امتیاز
0
#10
پاسخ : دنباله

لطفا تاپیکو تعطیل نکنید و سوالات دنباله رو در همین تاپیک بگذارید.
 

Wight

New Member
ارسال ها
1,188
لایک ها
888
امتیاز
0
#11
لایک ها amin

MR.Amin

New Member
ارسال ها
594
لایک ها
202
امتیاز
0
#12
پاسخ : دنباله

با تجزیه میشه (n-17) * (n+11)

پس از جمله اول تا جمله 16 ام منفیه....جمله 17 ام صفر و مابقی جملات مثبت اند.....پس در کل 16 جمله منفی داره.....

---- دو نوشته به هم متصل شده است ----

با تجزیه میشه (n-17) * (n+11)

پس از جمله اول تا جمله 16 ام منفیه....جمله 17 ام صفر و مابقی جملات مثبت اند.....پس در کل 16 جمله منفی داره.....
 

Wight

New Member
ارسال ها
1,188
لایک ها
888
امتیاز
0
#14
پاسخ : دنباله

ممنون!

در دنباله 4n^4-432n چندمین جمله کمترین مقدار را دارد؟
 
آخرین ویرایش توسط مدیر
لایک ها amin

amin

New Member
ارسال ها
312
لایک ها
515
امتیاز
0
#15
پاسخ : دنباله

با توجه به رفتار دنباله می توان دریافت که دنباله تا یک n مشخصی نزولی و سپس صعدوی خواهد بود.
 

Wight

New Member
ارسال ها
1,188
لایک ها
888
امتیاز
0
#16
پاسخ : دنباله

با توجه به رفتار دنباله می توان دریافت که دنباله تا یک n مشخصی نزولی و سپس صعدوی خواهد بود.
دنباله جدید که ویرایش کردم ! رو هم می شه جواب بدید!
 

Al!R3ZA

Well-Known Member
ارسال ها
1,903
لایک ها
3,166
امتیاز
113
#18
پاسخ : دنباله

تو پیدا کردن ماکسیمم و مینیمم ، مشتق هم خیلی کارسازه !
از رابطه دنباله مشتق بگیرید مساوی صفر بذارید و n رو پیدا کنید و تو رابطه دنباله بذارید
منتهی چون دنبالست ، اگه طبیعی در نیومد ، نزدیک ترین عدد طبیعی به اون عدد بدست اومده رو بذارید. اگه دقیقاً وسط دراومد ، دو طرفشو چک کنید. ( مثلاً برای 2.5 ، هم 2 رو چک کنید هم 3 رو )
 
بالا